4
$\begingroup$

I am struggling for quite some time, because of a problem involving Radon-Nikodým derivatives. I will try to describe the main features and perhaps somebody has an idea how to solve it.

I consider two sequences of measures on some compact set of $\mathbb{R}^2$: $\Xi_n$ and $\Lambda_n$ which are absolutely continuous in the sense $\Xi_n \ll \Lambda_n \ll \mathrm{Leb}$. I know that $\Lambda_n$ converges to some measure $\Lambda$ which is not absolutely continuous wrt. to Lebesgue measure anymore. (I suspect that it has a non-zero absolutely continuous part, but I am not quite sure how to prove this.) Furthermore, $\Xi_n$ also converges to some measure $\Xi$ satisfying $\Xi \ll \Lambda$.

The weird thing is that I need to integrate $\dfrac{d\Xi}{d\Lambda}$ against Lebsegue measure. In the prelimit case, there is no problem, because $\Xi_n$ and $\Lambda_n$ both have densities $\xi_n > 0$ and $\lambda_n > 0$ with respect to $\mathrm{Leb}$, so that simply $$ \dfrac{d\Xi_n}{d\Lambda_n} = \dfrac{\xi_n}{\lambda_n} \quad \mathrm{Leb}-\text{a.e.}\quad\text{(because $\lambda_n > 0$)} $$ Finally, $0\leq \dfrac{\xi_n}{\lambda_n}\leq 1$ so that the measure given by $d\kappa_n(x) := \dfrac{\xi_n(x)}{\lambda_n(x)}\cdot d\mathrm{Leb}(x)$ so that $(\kappa_n)$ is tight. For the moment, suppose that there is only one limiting point, i.e. $\kappa_n \to \kappa$, where $\kappa$ is some finite measure. I am wondering if one could conclude that $$ d\kappa(x) = \dfrac{d\Xi}{d\Lambda}(x)\cdot d\mathrm{Leb}. $$ In particular that would imply that $\Xi$ and $\Lambda$ have a non trivial absolutely continuous part. (Note that we may assume that $\mathrm{Leb}\ll\Lambda$ so that the Radon-Nikodým derivative is defined $\mathrm{Leb}$-a.e.)

I am not deep into measure theory, so I am really struggling with this problem. Especially because I find it strange to integrate the Radon-Nikodým derivative of $\Xi$ wrt. to $\Lambda$ against Lebesgue measure. I would be glad for any ideas or references that might go into this direction.

To give a larger context: I am working with a sequence $\Lambda_n(t)$ of measure-valued stochastic process that converges to a white noise process $\Lambda(t)$. And now I am looking at a related process $\Xi_n(t)$ and want to understand its limiting behaviour. So I would also be very glad about any ideas or references wrt. absolutely continuous parts of random fields and random measures wrt. to Lebesgue measure.

Edit: The convergence takes place at least in the distributional sense. I think I can also get it for Lipschitz continuous functions so that it is weak convergence.

Edit2: Initially, the integral $$ \int \dfrac{\xi_n}{\lambda_n}(x) \phi_n(x) dx $$ for some sequence of continuous (or smooth if you like) functions converging pointwise to some continuous (or smooth) limit phi appears in the problem. Since I could not figure out what will happen, I translated it towards the Radon-Nikodym derivative, hoping that since I require only weak convergence, the measure theoretic setting would be more helpful.

$\endgroup$
9
  • $\begingroup$ I'm curious: what do you need to integrate $d\Xi/d\Lambda$ for? Either I missed it or you left it out on purpose. Note that $d\Xi/d\Lambda$ is only defined a.s. up to $\Lambda$-nullsets, so you may obtain different results when integrating against Lebesgue measure depending on which version you choose. $\endgroup$
    – S.Surace
    Jul 9, 2020 at 13:01
  • 1
    $\begingroup$ In what sense does $\Lambda_n$ "converge" to $\Lambda$? $\endgroup$
    – Nik Weaver
    Jul 9, 2020 at 13:01
  • $\begingroup$ @S.Surace The integral with respect to Lebesgue measure appears in the infinitesimal generator of the process I am considering. Since $\Lambda$ can be decomposed into an absolutely continuous part and a singular part wrt. Lebesgue measure, I was thinking that a) for the absolutely continuous part, there is no problem and b) the singular part only changes things on a Lebesgue-nullset, so that different versions of this derivative shouldn't yield different results... $\endgroup$
    – Mushu Nrek
    Jul 9, 2020 at 13:55
  • $\begingroup$ @NikWeaver I edited my post. It is convergence in the distributional sense or weak convergence of measures. $\endgroup$
    – Mushu Nrek
    Jul 9, 2020 at 13:56
  • 1
    $\begingroup$ You still have the problem that $d\Xi/d\Lambda$ is defined only up to $\Lambda$-nullsets, giving potentially very different results when integrated against Lebesgue measure (some versions are not even integrable). For example, take $\Xi_n$ and $\Lambda_n$ each to be a weighted sum of a Gaussian centered at $0$ with variance $1/n$, and a uniform distribution on $[1,2]$. Then $d\Xi/d\Lambda$ may take arbitrary values on $(-\infty,0)\cup(0,1)\cup(2,\infty)$, which is a $\Lambda$ but not a Lebesgue nullset. Maybe try to multiply by indicator of $\text{supp}(\Lambda)$. $\endgroup$
    – S.Surace
    Jul 21, 2020 at 8:21

1 Answer 1

2
+100
$\begingroup$

There is no reason for the limit measure $\kappa$ to be related in any way to the limit measures $\Xi$ and $\Lambda$ (and, in particular, to their Radon-Nikodym derivative).

More precisely, if your sequences $\Xi_n,\Lambda_n$ on a compact $X\subset\mathbb R^2$ are such that $\Lambda=\lim\Lambda_n$ is singular with respect to $\text{Leb}$, then for any prescribed measure $\kappa$ on $X$ there are sequences $$ \Xi'_n\ll\Lambda'_n\ll \text{Leb} $$ with $$ \|\Xi_n-\Xi'_n\|, \|\Lambda_n-\Lambda'_n\|\to 0 $$ (so that, in particular, $\Xi'_n\to\Xi, \Lambda'_n\to\Lambda$), and such that the measures $$ \kappa'_n = \frac{d\Xi'_n}{d\Lambda'_n}\,\text{Leb} $$ weakly converge to $\kappa$. In fact, the presence of an ambient Euclidean space is completely irrelevant here, and instead of the Lebesgue measure one can talk about any reference measure on $X$.

The idea of the construction is very simple (I skip the details). Since $\lim\Lambda_n$ is singular with respect to the measure $\text{Leb}$, there are subsets $X_n\subset X$ with $\text{Leb}(X_n)\to\text{Leb}(X)$, whereas $\Xi(X_n),\Lambda_n(X_n)\to 0$. Fix a sequence $\epsilon_n\to 0$. Then the restrictions of $\Xi'_n$ and $\Lambda'_n$ to $X\setminus X_n$ are the multiples of $\Xi_n|_{X\setminus X_n}$ and $\Lambda_n|_{X\setminus X_n}$, respectively, chosen in such a way that $\Xi'_n(X\setminus X_n)=\Lambda'_n(X\setminus X_n)=1-\epsilon_n$, whereas on $X_n$ the measures $\Xi'_n$ and $\Lambda'_n$ can be defined in such a way that the Lebesgue measure multiplied by their ratio converges to $\kappa$.

$\endgroup$
1
  • $\begingroup$ Thank you very much! I see that in general, there is no way of getting a positive result. But in my case, I know (strongly suspect) that Lebesgue measure is absolutely continuous wrt $\Lambda$. This means that $\Lambda$ and Lebesgue are not completely singular, and in particular, I don't see how in that case, we can find the sets $X_n$ you mention. It would be great if you could elaborate on that. $\endgroup$
    – Mushu Nrek
    Jul 24, 2020 at 7:54

Your Answer

By clicking “Post Your Answer”, you agree to our terms of service and acknowledge you have read our privacy policy.

Not the answer you're looking for? Browse other questions tagged or ask your own question.